3
$\begingroup$

In what condition,

$\|u_k\|_{L^\infty(0,1)}\le C$ and

$u_{k}\rightarrow0$ strongly in $L^p(0,1),\;\forall 1<p<\infty$

imply that there exists a subsequence

$u_{k_i}\rightarrow0$ strongly in $L^\infty(0,1)$?

$\endgroup$
5
  • $\begingroup$ I cannot think of any such condition that isn't trivially equivalent to the conclusion. $\endgroup$ Sep 2, 2012 at 6:47
  • $\begingroup$ I edited the question by adding the strongly convergent conditon in $L^p$. $\endgroup$
    – jany
    Sep 2, 2012 at 6:59
  • $\begingroup$ Well, given that $\lVert u_k\rVert_{L^\infty}=\lim_{p\to\infty}\lVert u_k\rVert_{L^p}$, a suitable uniformity condition on the assumed $L^p$ convergence will do the trick, but once more, I'd consider that trivially equivalent to the conclusion. But by all means, I'd be quite happy to be proved wrong. $\endgroup$ Sep 2, 2012 at 7:11
  • $\begingroup$ Yes,I agree. But I wonder that whether there are some other conditions? Thank you so much. $\endgroup$
    – jany
    Sep 2, 2012 at 7:21
  • $\begingroup$ Equicontinuity of $u_n$ would suffice, though this does not fit so naturally with $L^p$ spaces. I am sure that you will not find any condition that implies convergence of a subsequence without also implying convergence of the entire sequence. $\endgroup$
    – user31373
    Sep 8, 2012 at 0:32

0

You must log in to answer this question.

Browse other questions tagged .